mustachegirl311
mustachegirl311
15.01.2021 • 
Mathematics

At the gym, Jasper was able to bench press 224 pounds, which was StartFraction 7 over 8 EndFraction of the weight that Balin was able to bench press. Which shows the correct equation and value of x, the weight that Balin could bench press?

Solved
Show answers

Ask an AI advisor a question